1.2 集合间的基本关系

\({\color{Red}{欢迎到学科网下载资料学习 }}\)
【基础过关系列】2022-2023学年高一数学上学期同步知识点剖析精品讲义(人教A版2019)
\({\color{Red}{ 跟贵哥学数学,so \quad easy!}}\)

必修第一册同步巩固,难度2颗星!

基础知识

子集

1 概念
对于两个集合\(A,B\),如果集合\(A\)的任何一个元素都是集合\(B\)的元素,我们说这两个集合有包含关系,称集合A是集合\(B\)的子集\((subset)\).
(感觉就像那些富二代跟我这些负二代说的一样:你有的我都有,你没的我也有)
记作:\(A⊆B\)(或\(B⊇A\)),读作:\(A\)包含于\(B\),或\(B\)包含\(A\).
当集合\(A\)不包含于集合\(B\)时,记作(\(A⊈B\)\(B⊉A\)).

2 Venn图

 

【例】 已知集合\(M=\{x\in Z |-1≤x<3\}\)\(N=\{x|x=|y|,y∈M\}\),判断集合\(M,N\)的关系.
解析 \(∵x∈Z\),且\(-1≤x<3\)
\(∴x\)的可能取值为\(-1,0,1,2\)
\(∴M=\{-1,0,1,2\}\)
\(∵y∈M\)\(∴|y|\)分别是\(0,1,2\)
\(∴N=\{0,1,2\}\)\(∴N⊆M\)
 

【练】若集合\(A=\{x|x≥0\}\),且\(B⊆A\),则集合\(B\)可能是 (  )
A.\(\{1,2\}\) \(\qquad\)\(\qquad\) B.\(\{x|x≤1\}\) \(\qquad\)\(\qquad\) C.\(\{-1,0,1\}\) \(\qquad\)\(\qquad\) D.\(R\)
解析 因为集合集合\(A=\{x|x≥0\}\),且\(B⊆A\),所以集合\(B\)是集合\(A\)的子集,
当集合\(B=\{1,2\}\)时,满足题意,
当集合\(B=\{x|x≤1\}\)时,\(-1∉A\),不满足题意,
当集合\(B=\{-1,0,1\}\)时,\(-1∉A\),不满足题意,
当集合\(B=R\)时,\(-1∉A\),不满足题意,
故选\(A\)

真子集

概念 :若集合\(A⊆B\),但存在元素\(x∈B\)\(x∉A\),则称集合\(A\)是集合\(B\)的真子集.
记作 \(A⊂B\)(或\(B⊃A\)) (有些地方\(⊂\)\(⫋\)\(⊊\)表示)
读作\(A\)真包含于\(B\)(或\(B\)真包含\(A\))
类比 \(⊆\)\(⊂\)的关系就好比\(≤\)与小于\(<\)的关系,\("≤"\)是小于或等于,\("⊆"\)是真包含或相等;
Eg \(3≤3\)是对的,而\(3<3\)是错的,若\(a<b\),则\(a≤b\)也成立;
对比下,\(A⊆A\)是对的,但\(A⊂A\)是错的,若\(A⊂B\),则\(A⊆B\)也成立.
 

【例】 \(\{1,2\}⊊A⊆\{1,2,3,4,5\}\),则满足条件的集合\(A\)的个数是(  )
A.\(6\) \(\qquad\)\(\qquad\) B.\(8\) \(\qquad\)\(\qquad\) C.\(7\)\(\qquad\)\(\qquad\) D.\(9\)
解析 \(∵\{1,2\}⊊A⊆\{1,2,3,4,5\}\)
\(∴\)集合\(A\)中除了含有\(1,2\)两个元素以外,至少必须含有另外一个元素,
因此满足条件的集合\(A\)\(\{1,2,3\}\)\(\{1,2,4\}\)\(\{1,2,5\}\)\(\{1,2,3,4\}\)\(\{1,2,3,5\}\)\(\{1,2,4,5\}\)\(\{1,2,3,4,5\}\)\(7\)个.故选:\(C\)
 

【练】已知集合\(A⊊\{0,1,2\}\),且\(A\)中至少含有一个奇数,则这样的集合\(A\)\(\underline{\quad \quad}\)个.
解析 \(∵\)集合\(A⊊\{0,1,2\}\)\(∴A=∅\)\(\{0\}\)\(\{1\}\)\(\{2\}\)\(\{0,1\}\)\(\{0,2\}\)\(\{1,2\}\)
\(∵A\)中至少含有一个奇数,\(∴A=\{1\},\{0,1\},\{1,2\}\)
\(∴\)这样的集合\(A\)\(3\)个.故选\(A\)

集合相等

如果\(A\)是集合\(B\)的子集,且集合\(B\)是集合\(A\)的子集,则集合\(A\)与集合\(B\)相等.即\(A⊆B\)\(B⊆A⇔A=B\).
正所谓“你中有我,我中有你,天人合一了”.
 

【例】 如果\(S=\{x|x=2n+1,n∈Z\}\)\(T=\{x|x=4k±1,k∈Z\}\),那么(  )
A.\(S\)真包含于\(T\) \(\qquad\)\(\qquad\) B.\(T\)真包含于\(S\) \(\qquad\)\(\qquad\) C.\(S=T\) \(\qquad\)\(\qquad\) D.\(S\)\(T\)没有交集
解析\(n\)为偶数,设\(n=2k\)\(k∈Z\),则\(x=2n+1=4k+1\)
\(n\)为奇数,设\(n=2k-1\)\(k∈Z\),则\(x=2n+1=4k-2+1=4k-1\)
\(∴\)集合\(S\)\(T\)的元素相同,\(∴S=T\).故选:\(C\)
 

【练】集合\(M=\{x|x=4k+2,k∈Z\}\)\(N=\{x|x=2k,k∈Z\}\)\(P=\{x|x=4k-2,k∈Z\}\),则\(M,N,P\)的关系(  )
A.\(M=P⊆N\) \(\qquad\)\(\qquad\) B.\(N=P⊆M\) \(\qquad\)\(\qquad\) C.\(M=N⊆P\)\(\qquad\)\(\qquad\) D.\(M=P=N\)
解析 \(M=\{…,-6,-2,2,6,10,14,18,22,26,…\}\)
\(N=\{…,-6,-4,-2,0,2,4,6,8,10,…\}\)
\(P=\{…,-6,-2,2,6,10,14,18,22,26,…\}\)
\(M=P⊆N\),故选\(A\).

几个结论

① 空集是任何集合的子集:\(∅⊆A\)
② 空集是任何非空集合的真子集;
③ 任何一个集合是它本身的子集;
④ 对于集合\(A ,B ,C\),如果\(A⊆B\)\(B⊆C\),那么\(A⊆C\)
⑤ 集合中有\(n\)个元素,则子集的个数为\(2^n\),真子集的个数为\(2^n-1\).(这个跟高二的二项式定理有关)
 

【例】 求集合\(A=\{1,2,3\}\)的子集和真子集.
解析 集合\(A=\{1,2,3\}\)的子集是\(∅,\{1\},\{2\},\{3\},\{1,2\}\),\(\{1,3\},\{2,3\},\{1,2,3\}\),共\(8\)个;
集合\(A=\{1,2,3\}\)的子集是\(∅,\{1\},\{2\},\{3\},\{1,2\}\),\(\{1,3\},\{2,3\}\),共\(7\)个.
 

【练】 求集合\(A=\{x|x^2-x-2=0\}\)的子集和真子集.
解析 集合\(A=\{x│x^2-x-2=0\}=\{-1,2\}\)
集合\(A=\{-1,2\}\)的子集是\(∅,\{-1\},\{2\},\{-1,2\}\),共\(4\)个;
集合\(A=\{-1,2\}\)的子集是\(∅,\{-1\},\{2\}\),共\(3\)个.

基本方法

【题型1】判断集合间的关系

【典题1】 指出下列各对集合之间的关系:
(1) \(A=\{x|x^2-1=0\}\)\(B=\{(x,y)|y=x^2-1\}\)
(2)\(A=\{x|x\)是菱形\(\}\)\(B=\{x|x\)是平行四边形\(\}\)
(3)\(A=\{x|x^2-3x-4<0\}\)\(B=\{x|x-5<0\}\)
(4)\(M=\{x|x=3n-1,n∈Z\}\)\(N=\{x|x=3n+2,n∈Z\}\)
解析 (1) 集合\(A\)的代表元素是数,集合\(B\)的代表元素是抛物线\(y=x^2-1\)上的点,故\(A\)\(B\)之间无包含关系.
(2) 菱形是特殊的平行四边形,故\(A⫋B\)
(3) 集合\(A=\{x│x^2-3x-4<0\}=\{x│-1<x<4\}\)\(B=\{x|x<5\}\)
用数轴表示集合\(A,B\)如图所示,由图可知\(A⫋B\)
image.png
(4)由列举法知\(M=\{…,-4,-1,2,5,8,…\}\)\(N=\{…,-4,-1,2,5,8,…\}\),故\(N=M\)
点拨 先分析集合元素的特征,若元素不同类肯定无包含关系,若元素同类则化简集合后再由判断.
 

【典题2】 集合\(M=\{x|x=4k+2,k∈Z\}\)\(N=\{x|x=2k,k∈Z\}\)\(P=\{x|x=4k-2,k∈Z\}\),则\(M,N,P\)的关系(  )
A.\(M=P⊆N\) \(\qquad\)\(\qquad\) B.\(N=P⊆M\) \(\qquad\)\(\qquad\) C.\(M=N⊆P\) \(\qquad\)\(\qquad\) D.\(M=P=N\)
解析 方法1 把每个集合用列举法表示, \(M=\{…,-6,-2,2,6,10,14,18,22,26,…\}\)
\(N=\{…,-6,-4,-2,0,2,4,6,8,10,…\}\)\(P=\{…,-6,-2,2,6,10,14,18,22,26,…\}\)
\(M=P⊆N\),故选\(A\).
方法2 \(x_1=4k_1+2∈M\)\(x_2=2k_2∈N\)\(x_3=4k_3-2∈P\),其中\(k_i∈Z\)
\(x_1=4k_1+2=4(k_1+1)-2∈P\),则\(M⊆P\)
\(x_3=4k_3-2=4(k_3-1)+2∈M\),则\(P⊆M\),故\(M=P\).
\(x_1=4k_1+2=2(2k_1+1)∈N\),则\(M⊆N\)
\(x_{2}=2 k_{2}=4 \times \dfrac{k_{2}-1}{2}+2\)不一定属于\(M\)
\(M=P⊆N\),故选\(A\).
方法3 集合\(M\)\(P\)的元素均是被\(4\)除余\(2\),则\(M=P\);集合\(N\)的元素是偶数,而集合\(M\)元素是\(2×\)奇数.
点拨 方法1利用列举法通过观察确定集合关系,不够严谨;方法2利用严谨的推理得到关系,略显抽象;方法3从“余数”的角度处理较为简洁.

巩固练习

1.以下六个写法中:①\(\{0\}∈\{0,1,2\}\);②\(∅⊆\{1,2\}\);③\(∅∈\{0\}\);④\(\{0,1,2\}=\{2,0,1\}\);⑤\(0∈∅\);正确的个数有(  )
A.\(1\)\(\qquad\)\(\qquad\) B.\(2\)\(\qquad\)\(\qquad\) C.\(3\)\(\qquad\)\(\qquad\) D.\(4\)
 

2.指出下列各对集合之间的关系:
(1)\(A=\{-1,1\}\)\(B=\{(-1,-1),(-1,1),(1,-1),(1,1)\}\)
(2)\(A=\{x|x\)是等边三角形\(\}\)\(B=\{x|x\)是等腰三角形\(\}\)
(3)\(A=\{x|-1<x<4\}\)\(B=\{x|x-5<0\}\)
(4)\(M={x|x=2n-1,n∈N^*}\)\(N=\{x|x=2n+1,n∈N^*\}\)
 

3.已知集合\(M=\left\{x \mid x=\dfrac{k}{2}+\dfrac{1}{4}, k \in Z\right\}\),\(N=\left\{x \mid x=\dfrac{k}{4}+\dfrac{1}{2}, k \in Z\right\}\),若\(x_0∈M\),则\(x_0\)\(N\)的关系是(  )
A.\(x_0∈N\) \(\qquad\)\(\qquad\) B.\(x_0∉N\) \(\qquad\)\(\qquad\) C.\(x_0∈N\)\(x_0∉N\) \(\qquad\)\(\qquad\) D.不能确定
 

参考答案

  1. 答案 \(B\)
    解析 对于①:是集合与集合的关系,应该是\(\{0\}⊂\{0,1,2\}\)\(∴\)①不对.
    对于②:空集是任何集合的子集,应该是\(∅⊆\{1,2\}\)\(∴\)②对.
    对于③:\(∅\)是一个集合,是集合与集合的关系,\(∅⊂\{0\}\)\(∴\)③不对.
    对于④:根据集合的无序性可知\(\{0,1,2\}=\{2,0,1\}\)\(∴\)④对.
    对于⑤:\(∅\)是一个空集合,表示没有任何元素,应该是\(0∉∅\);∴⑤不对.
    正确的是:②④.
    故选:\(B\)

  2. 答案 (1) 无包含关系 (2) \(A⫋B\) (3)\(A⫋B\) (4) \(N⫋M\)
    解析 (1)集合\(A\)的代表元素是数,集合\(B\)的代表元素是有序实数对,
    \(A\)\(B\)之间无包含关系.
    (2)等边三角形是三边相等的三角形,等腰三角形是两边相等的三角形,故\(A⫋B\)
    (3)集合\(B=\{x|x<5\}\),用数轴表示集合\(A,B\)如图所示,由图可知\(A⫋B\)

    image.png
    (4)由列举法知\(M=\{1,3,5,7,…\}\)\(N=\{3,5,7,9,…\}\)
    \(N⫋M\)

  3. 答案 \(A\)
    解析 \(M=\left\{x \mid x=\dfrac{n}{2}+\dfrac{1}{4}=\dfrac{2 n+1}{4}, \quad n \in Z\right\}\),显然\(M\)的分子为奇数,\(N=\left\{x \mid x=\dfrac{k}{4}+\dfrac{1}{2}=\dfrac{k+2}{4}, \quad k \in Z\right\}\),显然\(N\)的分子为整数,
    \(∴\)集合\(M、N\)的关系为\(M⊊N\)\(∵x_0∈M\)\(∴x_0∈N\)
    故选:\(A\)

【题型2】求已知集合的子集或真子集

【典题1】 集合\(M=\{x|x∈Z\)\(\dfrac{12}{1+x} \in N\}\),则\(M\)的非空真子集的个数是(  )
A.\(30\)\(\qquad\)\(\qquad\) B.\(32\)\(\qquad\)\(\qquad\) C.\(62\)\(\qquad\)\(\qquad\) D.\(64\)
解析 由题意集合\(M=\{x|x∈Z\)\(\dfrac{12}{1+x} \in N\}\)\(=\{x|x=0,1,2,3,5,11\}\)
由对于含有\(n\)个元素的集合,利用公式\(2^n-2\)计算出\(M\)的非空真子集个数,
\(∴M\)的非空真子集的个数是\(2^6-2=62\),故选:\(C\)
点拨 先化简集合,确定集合的元素个数,利用公式求解.

巩固练习

1.集合\(\{a,b,c,d\}\)的非空真子集的个数(  )
A.\(16\)\(\qquad\)\(\qquad\) B.\(15\)\(\qquad\)\(\qquad\) C.\(14\)\(\qquad\)\(\qquad\) D.\(13\)
 

2.定义集合\(A*B=\{x|x∈A,\)\(x∉B\}\),若\(A=\{1,3,5,7\}\)\(B=\{2,3,5\}\),则\(A*B\)的子集个数为\(\underline{\quad \quad}\) .
 

3.已知集合\(B=\{(x,y)∣4x+3y-12<0,x∈N^*,y∈N^* \}\),则\(B\)的子集个数为\(\underline{\quad \quad}\).
 

参考答案

  1. 答案 \(C\)
    解析 \(∵\)集合\(\{a,b,c,d\}\)\(4\)个元素,\(∴\)则集合\(\{a,b,c,d\}\)\(2^4=16\)个子集,
    故集合\(\{a,b,c,d\}\)的非空真子集的个数为\(14\);故选\(C\)
  2. 答案 \(4\)
    解析 由题意:\(A*B=\{1,7\}\),故其子集为\(∅,\{1\},\{7\},\{1,7\}\),个数为\(4\).
  3. 答案 \(8\)
    解析 \(∵\)集合\(B=\{(x,y)∣4x+3y-12<0,x∈N^*,y∈N^* \}\)
    \(∴B=\{(1,1),(1,2),(2,1)\}\)
    \(∴B\)中含有\(3\)个元素,集合\(B\)的子集个数有\(2^3=8\)

【题型3】综合应用

【典题1】 已知集合\(A=\{x|x^2+x-6=0\}\)\(B=\{x|mx+1=0\}\),且\(B⊆A\),则实数\(m=\)\(\underline{\quad \quad}\).
解析 解方程:\(x^2+x-6=0\),得:\(x=-3\)\(x=2\),即集合\(A=\{-3,2\}\)
\(B⊆A\)
①当\(B=∅\),即\(m=0\)时,满足题意,
②当\(B=\{-3\}\),即\(-\dfrac{1}{m}=-3\),即\(m=\dfrac{1}{3}\)时,满足\(B⊆A\)
③当\(B=\{2\}\),即\(-\dfrac{1}{m}=2\),即\(m=-\dfrac{1}{2}\)时,满足\(B⊆A\)
综合①②③得:\(m=1\)\(m=\dfrac{1}{3}\)\(m=-\dfrac{1}{2}\)
点拨 注意不用漏了\(B=∅\)的情况;求解方程\(mx+1=0\),注意\(m=0\)的情况.
 

【典题2】已知集合\(A=\{x|x^2-3x-10≤0\}\)
(1)若\(B=\{x|m-6≤x≤2m-1\}\),\(A⊆B\),求实数\(m\)的取值范围;
(2)若\(B=\{x|m+1≤x≤2m-1\}\),\(B⊆A\),求实数\(m\)的取值范围.
解析集合\(A=\{x|x^2-3x-10≤0\}=\{x|-2≤x≤5\}\)
(1)\(∵A⊆B\)\(\therefore\left\{\begin{array}{l} m-6 \leq-2 \\ 2 m-1 \geq 5 \end{array}\right.\),解得:\(3≤m≤4\)
\(∴\)实数\(m\)的取值范围为\([3,4]\)
(2)\(∵B⊆A\)
①当\(B=∅\)时,\(m+1>2m-1\),即\(m<2\)
②当\(B≠∅\)时,\(\left\{\begin{array}{l} m+1 \leq 2 m-1 \\ m+1 \geq-2 \\ 2 m-1 \leq 5 \end{array}\right.\),解得:\(2≤m≤3\)
综上所述,实数\(m\)的取值范围为:\((-∞,3]\)
点拨 若集合\(A=\{x│m<x<n\}=∅\),即\(m≥n\);对于涉及不等式的集合关系,可画数轴了解其关系.

巩固练习

1.已知集合\(A=\{x∣a-1≤x≤a+2\}\)\(B=\{x∣3<x<5\}\),则能使\(A⊇B\)成立的实数\(a\)的取值范围是( )
A.\(\{a∣3<a≤4\}\) \(\qquad\)\(\qquad\) B.\(\{a∣3<a<4\}\) \(\qquad\)\(\qquad\) C.\(\{a∣3≤a≤4\}\) \(\qquad\)\(\qquad\) D.\(∅\)
 

2.若集合\(A=\{x∣x^2+x-6=0\}\)\(B=\{x∣mx+1=0\}\)\(B⊆A\),则\(m=\)\(\underline{\quad \quad}\)
 

3.已知集合\(A=\{x|x^2-3x-10≤0\}\),集合\(B=\{x|p+1≤x≤2p-1\}\).若\(B⊆A\),则实数\(p\)的取值范围是\(\underline{\quad \quad}\)
 

参考答案

  1. 答案 \(C\)
    解析 \(∵A⊇B\)\(\therefore\left\{\begin{array}{l} a-1 \leq 3 \\ a+2 \geq 5 \end{array}\right.\)\(∴3≤a≤4\),故选\(C\)
  2. 答案 \(C\)
    解析 \(A=\{x∣x^2+x-6=0\}=\{-3,2\}\)
    因为\(B⊆A\),所以方程\(mx+1=0\)的解可以是\(-3\)\(2\)或无解.
    \(mx+1=0\)的解为\(-3\)时,由\(-3m+1=0\)\(m=\dfrac{1}{3}\)
    \(mx+1=0\)的解为\(2\)时,由\(2m+1=0\)\(m=-\dfrac{1}{2}\)
    \(mx+1=0\)无解时,\(m=0\)
    综上可知,\(m\)的值为\(\dfrac{1}{3}\)\(-\dfrac{1}{2}\)\(0\)
  3. 答案 \((-∞,3]\)
    解析 \(∵\)集合\(A=\{x|x^2-3x-10≤0\}=\{x|-2≤x≤5\}\),集合\(B=\{x|p+1≤x≤2p-1\}\)
    \(B⊆A\)
    \(∴\)\(B=∅\)时,\(p+1>2p-1\)\(p<2\)
    \(B≠∅\)时,有\(-2≤p+1\)\(p+1≤2p-1\)\(2p-1≤5\),解得\(2≤p≤3\)
    综上,\(p\)的范围为\((-∞,3]\)

分层练习

【A组---基础题】

1.设\(A,B\)是两个集合,有下列四个结论:
①若\(A⊈B\),则对任意\(x∈A\),有\(x∉B\)
②若\(A⊈B\),则集合\(A\)中的元素个数多于集合\(B\)中的元素个数;
③若\(A⊈B\),则\(B⊈A\)
④若\(A⊈B\),则一定存在\(x∈A\),有\(x∉B\)
其中正确结论的个数为(  )
A.\(4\) \(\qquad\)\(\qquad\) B.\(3\) \(\qquad\)\(\qquad\) C.\(2\) \(\qquad\)\(\qquad\) D.\(1\)
 

2.已知全集\(U=R\),则正确表示集合\(M=\{-1,0,1\}\)\(N=\{x∣x^2+x=0\}\)关系的韦恩(Venn)图是 ( )
image.png
 

3.满足条件\(\{1,2,3,4\}⊆M⊊\{1,2,3,4,5,6\}\)的集合\(M\)的个数是(  )
A.\(2\) \(\qquad\)\(\qquad\) B.\(3\) \(\qquad\)\(\qquad\) C.\(4\) \(\qquad\)\(\qquad\) D.\(5\)
 

4.集合\(M=\{x|x=3k-2,k∈Z\}\)\(P=\{y|y=3n+1,n∈Z\}\)\(S=\{z|z=6m+1,m∈Z\}\)之间的关系是(  )
A.\(S\)真包含于\(P\)真包含于\(M\)
B.\(S=P\)真包含于\(M\)
C.\(S\)真包含于\(P=M\)
D.\(M=P\)真包含于\(S\)
 

5.已知\(M=\left\{x \in N \mid \dfrac{6}{6-x} \in N\right\}\),则集合\(M\)的子集的个数是(  )
A.\(8\) \(\qquad\)\(\qquad\) B.\(16\) \(\qquad\)\(\qquad\) C.\(32\) \(\qquad\)\(\qquad\) D.\(64\)
 

6.已知集合\(M=\{0,1\}\),集合\(N=\{0,2,1-m\}\),若\(M \subseteq N\),则实数\(m=\)\(\underline{\quad \quad}\)
 

7.含有三个实数的集合可表示为\(\left\{a, \dfrac{b}{a}, 1\right\}\),也可表示为\(\{a^2,a+b\}\),则\(a^{2013}+b^{2014}\)的值为\(\underline{\quad \quad}\) .
 

8.已知\(A=\{x|x<-1\)\(x>5\}\)\(B=\{x|a≤x<a+4\}\),若\(B⫋A\),则实数\(a\)的取值范围是\(\underline{\quad \quad}\)
 

9.集合\(A=\{-1,2\}\)\(B=\{x|ax-2=0\}\),若\(B⊆A\),则由实数\(a\)组成的集合为\(\underline{\quad \quad}\).
 

参考答案

  1. 答案 \(D\)
    解析 对于①,不一定,比如\(A=\{1,2,4\},B=\{1,2,3\}\).故错误;
    ②若\(A⊈B\),不一定,比如\(A=\{1,2,4\},B=\{1,2,3,5,6\}\).故错误;
    ③若\(B⊊A\),则\(A⊈B\),但\(B⊈A\)不成立,故错误;
    ④若\(A⊈B\),则一定存在\(x∈A\),有\(x∉B\),故正确.
    故正确结论的个数为\(1\)个,故选:\(D\)
  2. 答案 \(B\)
    解析\(N=\{x∣x^2+x=0\}=\{-1,0\}\),则\(N⊂M\).
  3. 答案 \(B\)
    解析 \(∵\{1,2,3,4\}⊆M⊊\{1,2,3,4,5,6\}\)
    \(∴\)集合\(M\)中必有元素\(1,2,3,4\)
    且集合\(M\)中还有元素\(5,6\)中的\(0\)个或\(1\)个,
    \(∴\)满足条件\(\{1,2,3,4\}⊆M⊊\{1,2,3,4,5,6\}\)的集合M的个数是:\(3\)
    故选:\(B\)
  4. 答案 \(C\)
    解析 \(∵M=\{x|x=3k-2,k∈Z\}\)\(N=\{y|y=3n+1,n∈Z\}\)\(S=\{y|y=6m+1,m∈Z\}\)
    \(∴M=\{…-8,-5,-2,1,4,7,10,13,16…\}\),
    \(P=\{…-8,-5,-2,1,4,7,10…\}\)\(S=\{…1,7,13,19,25,…\}\)
    \(S⊊P=M\),故选:\(C\)
  5. 答案 \(B\)
    解析 由于\(x∈N\),则\(x=0,1,2,3,…\)
    \(x=0\)时,\(\dfrac{6}{6-0}=1 \in N\),当\(x=1\)时,\(\dfrac{6}{6-1}=\dfrac{6}{5} \notin N\)
    \(x=2\)时,\(\dfrac{6}{6-2}=\dfrac{3}{2} \not \nsubseteq N\),当\(x=3\)时,\(\dfrac{6}{6-3}=\dfrac{6}{3}=2 \in N\),
    \(x=4\)时,\(\dfrac{6}{6-4}=\dfrac{6}{2}=3 \in N\),当\(x=5\)时,\(\dfrac{6}{6-5}=6 \in N\)
    \(x=7\)时,\(\dfrac{6}{6-7}=-1 \notin N\),…
    故集合集合\(A=\{0,3,4,5\}\),所以集合\(A\)子集个数为\(2^4=16\)个.
    故选:\(B\)
  6. 答案 \(0\)
    解析 由题意知\(M⊆N\),又集合\(M=\{0,1\}\),因此\(1∈N\),即\(1-m=1\).故\(m=0\)
  7. 答案 \(-1\)
    解析\(a=a^2\)\(a=0\)\(1\),代回集合,不满足集合元素的互异性;则\(a=a+b\),得\(b=0\)
    从而易得\(a=-1\);故\(a^{2013}+b^{2014}=-1\).
  8. 答案 \(\{a|a>5\)\(a≤-5\}\)
    解析 作出数轴可得,要使\(B⫋A\),则必须\(a+4≤-1\)\(a>5\),解之得\(\{a|a>5\)\(a≤-5\}\)
  9. 答案 \(\{-2,1,0\}\)
    解析 \(∵\)集合\(A=\{-1,2\}\)\(B=\{x|ax-2=0\}\)\(B⊆A\)
    \(∴B=∅\)\(B=\{-1\}\)\(B=\{2\}\)
    \(∴a=0,1,-2\)
    \(∴\)由实数\(a\)组成的集合为:\(\{-2,1,0\}\)

【B组---提高题】

1.集合\(A=\{(x,y)|y=|x|\}\),集合\(B=\{(x,y)|y>0,x∈R\}\),则下列说法正确的是(  )
A.\(A⊆B\) \(\qquad\)\(\qquad\) B.\(B⊆A\) \(\qquad\)\(\qquad\) C.\(A∩B=∅\) \(\qquad\)\(\qquad\) D.集合\(A、B\)间没有包含关系
 

2.设集合\(P_1=\{x|x^2+ax+1>0\}\)\(P_2=\{x|x^2+ax+2>0\}\),其中\(a∈R\),下列说法正确的是(  )
A.对任意\(a\)\(P_1\)\(P_2\)的子集
B.对任意\(a\)\(P_1\)不是\(P_2\)的子集
C.存在\(a\),使得\(P_1\)不是\(P_2\)的子集
D.存在\(a\),使得\(P_2\)\(P_1\)的子集
 

3.已知集合\(A=\{x|x=4n+1,n∈Z\}\)\(B=\{x|x=4n-3,n∈z\}\)\(C=\{x|x=8n+1,n∈z\}\),则\(A,B,C\)的关系是(  )
A.\(C\)\(B\)的真子集、\(B\)\(A\)的真子集
B.\(A\)\(B\)的真子集、\(B\)\(C\)的真子集
C.\(C\)\(A\)的真子集、\(A=B\)
D.\(A=B=C\)
 

4.已知集合\(M=\left\{x \mid x=m+\dfrac{1}{6}, m \in N\right\}\)\(N=\left\{x \mid x=\dfrac{n}{2}-\dfrac{1}{3}, n \in N\right\}\),则\(M,N\)的关系为(  )
A.\(M=N\) \(\qquad\)\(\qquad\) B.\(N⊊M\) \(\qquad\)\(\qquad\) C.\(M⊊N\) \(\qquad\)\(\qquad\) D.\(N⊆M\)
 

5.已知集合\(A=\left\{x \mid x=k+\dfrac{1}{6}, k \in N\right\}\)\(B=\left\{x \mid x=\dfrac{m}{2}-\dfrac{1}{3}, m \in N\right\}\)\(C=\left\{x \mid x=\dfrac{n}{2}+\dfrac{1}{6}, n \in N\right\}\),则集合\(A、B、C\)的大小关系是(  )
A.\(A⫋C⫋B\) \(\qquad\)\(\qquad\) B.\(C⫋A⫋B\) \(\qquad\)\(\qquad\) C.\(A⫋B=C\)\(\qquad\)\(\qquad\) D.\(A⫋B⫋C\)
 

6.已知集合\(A=\{x|x>1\}\),\(B=\{x|ax>1\}\),若\(B⊆A\),则实数\(a\)的取值范围(  )
A.\((0,1)\) \(\qquad\)\(\qquad\) B.\((0,1]\) \(\qquad\)\(\qquad\) C.\([0,1]\) \(\qquad\)\(\qquad\) D.\([0,1)\)
 

7.若集合\(A=\{x∣(k+2)x^2+2kx+1=0\}\)有且仅有\(2\)个子集,则实数\(k\)的值是 (  )
A.\(-2\) \(\qquad\)\(\qquad\) B.\(-2\)\(-1\) \(\qquad\)\(\qquad\) C.\(2\)\(-1\) \(\qquad\)\(\qquad\) D.\(±2\)\(-1\)
 

8.已知集合\(A=\left\{x \mid x^{2}-3 x+2=0\right\}\)\(B={x|x^2+2(a+1)x+a^2-5=0}\),若\(B⊆A\),则\(a\)的取值范围\(\underline{\quad \quad}\)
 

9.已知集合\(A=\{x|2a≤x≤a^2+1\}\)\(B=\{x|x^2-3(a+1)x+2(3a+1)≤0\}\),其中\(a∈R\)
(1)若\(4∈A,3∉A\),求实数\(a\)的取值范围;
(2)若\(A⊆B\),求实数\(a\)的取值范围.
 
 

参考答案

  1. 答案 \(D\)
    解析 \(A=\{(x,y)|y≥0,x∈R,\)\(y=|x|\}\)
    \(∴(0,0)∈A\),而\((0,0)∉B\)\((0,1)∈B\),而\((0,1)∉A\)
    \(∴\)集合\(A,B\)间没有包含关系.
    故选:\(D\)
  2. 答案 \(A\)
    解析\(x^2+ax+1>0\),则有\(x^2+ax+2=x^2+ax+1+1>0+1>0\)
    \(x^2+ax+2>0\),则有\(x^2+ax+1=x^2+ax+2-1>-1\)
    不能推出\(x^2+ax+1>0\)
    \(P_1⊊P_2\),故选:\(A\)
  3. 答案 \(C\)
    解析 \(∵A=\{x|x=4n+1,n∈Z\}\)\(B=\{x|x=4n-3=4(n-1)+1,n∈Z\}\)
    \(∴A=B\);故排除选项\(A,B\);又\(∵5∈A,5∉C\)\(∴\)排除\(D\)
    故选\(C\)
  4. 答案 \(C\)
    解析 由集合\(M=\left\{x \mid x=m+\dfrac{1}{6}, m \in N\right\}\)\(N=\left\{x \mid x=\dfrac{n}{2}-\dfrac{1}{3}, n \in N\right\}\),用\(n\)代替\(n+1\)可得\(\dfrac{3(n+1)-2}{6}=\dfrac{3 n+1}{6}, \quad n \in Z\)
    所以\(N=\left\{x \mid x=\dfrac{3 n+1}{6}, \quad n \in Z\right\}\),所以\(M⊊N\),故选:\(C\)
  5. 答案 \(A\)
    解析 \(∵\)集合\(C=\left\{x \mid x=\dfrac{n}{2}+\dfrac{1}{6}, n \in N\right\}\)
    \(∴\)\(n=2a(a∈N)\)时,\(x=\dfrac{2 a}{2}+\dfrac{1}{6}=a+\dfrac{1}{6}\)
    \(n=b-1(b∈N^*)\)时,\(x=\dfrac{b-1}{2}+\dfrac{1}{6}=\dfrac{b}{2}-\dfrac{1}{2}+\dfrac{1}{6}=\dfrac{b}{2}-\dfrac{1}{3}\)
    \(∵\)集合\(A=\left\{x \mid x=k+\dfrac{1}{6}, \quad k \in N\right\}\)\(∴A⫋C\)
    \(∵\)集合\(B=\left\{x \mid x=\dfrac{m}{2}-\dfrac{1}{3}, \quad m \in N\right\}\)
    \(∴\)集合\(B\)比集合\(C\)多一个元素\(=-\dfrac{1}{3}\),即\(C⫋B\)
    综上所求:\(A⫋C⫋B\)
    故选:\(A\)
  6. 答案 \(C\)
    解析 已知集合\(A=\{x|x>1\},B=\{x|ax>1\}\)
    \(B⊆A\),则\(A\)集合包含\(B\)集合的所以元素,
    \(B\)集合时,当\(a<0\)时,不满足题设条件,
    \(a=0\)时,\(x\)无实数解,\(B\)集合为空集,满足条件,
    \(a>0\)时,\(x>\dfrac{1}{a}\),则\(\dfrac{1}{a} \geq 1, a \leq 1\),即\(0<a≤1\)
    综上则实数\(a\)的取值范围为:\([0,1]\),
    故选:\(C\)
  7. 答案 \(D\)
    解析 要使得一个集合有且仅有\(2\)个子集,则须使集合有且仅有\(1\)个元素.
    因此方程\((k+2)x^2+2kx+1=0\)要么有且仅有一个实根,即\(k+2=0,k=-2\)
    要么有且仅有两个相等的实根.由\(Δ=(2k)^2-4(k+2)=0\)\(k=-1\)\(k=2\).
    所以选\(D\)
  8. 答案 \((-∞,-3]\)
    解析 由题得\(A=\{1,2\}\),因为\(B⊆A\),则
    \(B=∅\),所以\(△=4(a+1)^2-4(a^2-5)=8a+24<0\),解得\(a<-3\)
    \(B=\{1\}\),则\(\left\{\begin{array}{l} \Delta=8 a+24=0 \\ 1+2(a+1)+a^{2}-5=0 \end{array}\right.\),解得方程组无解;
    \(B=\{2\}\),则\(\left\{\begin{array}{l} \Delta=8 a+24=0 \\ 4+4(a+1)+a^{2}-5=0 \end{array}\right.\),解得\(a=-3\)
    \(B=\{1,2\}\),则\(\left\{\begin{array}{l} \Delta=8 a+24>0 \\ 1+2(a+1)+a^{2}-5=0 \\ 4+4(a+1)+a^{2}-5=0 \end{array}\right.\),无解.
    综上\(a∈(-∞,-3]\)
  9. 答案 (1) \([\sqrt{3}, 2]\) (2)\(\{a|a=-1\)\(1≤a≤3\}\)
    解析 (1)因为\(4∈A\),所以\(2a≤4≤a^2+1\),解得\(a \leq-\sqrt{3}\)\(\sqrt{3} \leq a \leq 2\)
    \(3∉A\),所以\(2a>3\)\(a^2+1<3\),故\(-\sqrt{2}<a<\sqrt{2}\)\(a>\dfrac{3}{2}\)
    \(∴\)\(4∈A\)\(3∉A\),有\(\sqrt{3} \leq a \leq 2\)
    \(a\)的取值范围是:\([\sqrt{3}, 2]\)
    (2)\(B=\{x|(x-2)[x-(3a+1)]≤0\}\)
    \(3a+1=2\),即\(a=\dfrac{1}{3}\)时,\(B=\{2\}\),不合题意.
    \(3a+1<2\),即\(a<\dfrac{1}{3}\)时,\(B={x|3a+1≤x≤2}\),所以\(\left\{\begin{array}{l} 3 a+1 \leq 2 a \\ a^{2}+1 \leq 2 \end{array}\right.\)
    \(\therefore\left\{\begin{array}{l} a \leq-1 \\ -1 \leq a \leq 1 \end{array}\right.\),解得\(a=-1\)
    \(3a+1>2\),即\(a>\dfrac{1}{3}\)时,\(B=\{x|2≤x≤3a+1\}\),所以\(\left\{\begin{array}{l} 2 a \geq 2 \\ a^{2}+1 \leq 3 a+1 \end{array}\right.\)
    \(\therefore\left\{\begin{array}{l} a \geq 1 \\ 0 \leq a \leq 3 \end{array}\right.\),解得\(1≤a≤3\)
    综上知,\(a=-1\)\(1≤a≤3\)
    故实数\(a\)的取值范围是\(\{a \mid a=-1\)\(1≤a≤3\}\)

【C组---拓展题】

1.设集合\(M=\{1,2,3,4,5,6\}\)\(S_1,S_2,⋯,S_k\)都是\(M\)的含两个元素的子集,且满足:对任意的\(S_i=\{a_i,b_i \}\)\(S_j={a_j,b_j }(i≠j,i、j∈{1,2,3,…,k})\),都有\(\min \left\{\dfrac{a_{i}}{b_{i}}, \dfrac{b_{i}}{a_{i}}\right\} \neq \min \left\{\dfrac{a_{j}}{b_{j}}, \dfrac{b_{j}}{a_{j}}\right\}(\min \{x, y\}\)表示两个数\(x,y\)中的较小者\()\),则\(k\)的最大值是( )
A.\(10\) \(\qquad\)\(\qquad\) B.\(11\) \(\qquad\)\(\qquad\) C.\(12\)\(\qquad\)\(\qquad\) D.\(13\)
 

2.已知\(A=\{x|x^2-5x+4≤0\},B=\{x|x^2-2ax+a+2≤0\}\),且\(B⊆A\),则\(a\)的取值范围为(  )
A. \(\left[2, \dfrac{18}{7}\right]\) \(\qquad\)\(\qquad\) B. \(\left(-1, \dfrac{18}{7}\right]\) \(\qquad\)\(\qquad\) C. \(\left(-\infty, \dfrac{18}{7}\right]\) \(\qquad\)\(\qquad\) D.\([2,+∞)\)
 

3.设\(S=\{r_1,r_2,…,r_n\}⊆\{1,2,3,…,50\}\),且\(S\)中任意两数之和不能被\(7\)整除,则\(n\)的最大值为\(\underline{\quad \quad}\)
 

参考答案

  1. 答案 \(B\)
    解析\(2\)个元素的子集有\(15\)个,但\(\{1,2\}、\{2,4\}、\{3,6\}\)只能取一个;\(\{1,3\}、\{2,6\}\)只能取一个;\(\{2,3\}、\{4,6\}\)只能取一个,故满足条件的两个元素的集合有\(11\)个.
  2. 答案\(B\)
    解析由题意:\(A=\left\{x \mid x^{2}-5 x+4 \leq 0\right\}=\{x \mid 1 \leq x \leq 4\}\),\(B=\left\{x \mid x^{2}-2 a x+a+2 \leq 0\right\}\)
    \(∵B⊆A\),
    \(∴\)\(B=∅\)时,满足题意,
    此时\(x^2-2ax+a+2≤0\)无解,\(△<0,4a^2-4(a+2)<0\),解得\(-1<a<2\)
    \(B≠∅\)时,要使\(B⊆A\)成立,此时令\(f(x)=x^2-2ax+a+2≤0\)有解,
    \(△≥0\),\(4a^2-4(a+2)≥0\),解得:\(a≥2\)\(a≤-1\)
    根据二次函数根的分布,可得\(\left\{\begin{array}{l} f(1) \geq 0 \\ f(4) \geq 0 \end{array}\right.\),即\(\left\{\begin{array}{l} 1-2 a+a+2 \geq 0 \\ 16-8 a+a+2 \geq 0 \end{array}\right.\),解得:\(a \leq \dfrac{18}{7}\)
    \(\therefore 2 \leq a \leq \dfrac{18}{7}\)
    综上可得:\(-1<a \leq \dfrac{18}{7}\),故选:\(B\).
  3. 答案 \(23\)
    解析 可将\(S\)集合分为6组
    \(S_0=\{7,14,21,28,35,42,49\}\),则\(card⁡(S_0 )=7\)
    \(S_1=\{1,8,15,22,29,36,43,50\}\),则\(card(S_1 )=8\)
    \(S_2=\{2,9,16,23,30,37,44\}\),则\(card⁡(S_2 )=7\)
    \(S_3=\{3,10,17,24,31,38,45\}\),则\(card⁡(S_3 )=7\)
    \(S_4=\{4,11,18,25,32,39,46\}\),则\(card⁡(S_4 )=7\)
    \(S_5=\{5,12,19,26,33,40,47\}\),则\(card⁡(S_5 )=7\)
    \(S_6=\{6,13,20,27,34,41,48\}\),则\(card⁡(S_6 )=7\)
    \(S\)中的任何两个数之和不能被\(7\)整除,
    \(S_1\)\(S_6\)\(S_2\)\(S_5\)\(S_3\)\(S_4\)中不能同时取数,且\(S_0\)中最多取一个,
    所以最多的取法是取\(S_1\)\(S_2(\)\(S_5)\)\(S_3(\)\(S_4)\),和S\(_0\)中的一个,
    \(card(S)_{max}=8+7+7+1=23\)
    故答案为\(23\).
posted @ 2022-08-30 10:01  贵哥讲数学  阅读(304)  评论(0编辑  收藏  举报
//更改网页ico // 实现数学符号与汉字间有间隙 //文章页加大页面,隐藏侧边栏